Question

In: Economics

Horizontal Mergers. Consider a market that initially has two firms selling differentiated products and competing in...

Horizontal Mergers. Consider a market that initially has two firms selling differentiated products and competing in prices. The cost of production of each firm is C(q)=0. Demand for the two goods is given by the following system: q1=420-4p1+p2 q2=420-4p2+p1 The goal of this question is to find by how much the prices of the goods will increase if the firms horizontally merge. To do this, we first derive the equilibrium prices before the merger, and then compare with the optimal prices set by the merged firm.

a) Firm 1’s profit maximization problem is to choose p1 so as to solve: max [p1 (420-4p1+p2)] p1≥0 Derive firm 1’s first order condition (differentiating this profit function with respect to p1).

b) Assuming in equilibrium p1=p2, find the equilibrium prices before the merger.

c) If the firms merge, and continue to sell the same two products, the merged firm chooses both prices choosing p1 and p2 so as maximize problem. Write the profit maximization problem.

d) Find the profit maximizing prices. You may assume that these prices are equal p1=p2.

Solutions

Expert Solution

In this question we are not given the costs, so we take total revenue as the sole profit criteria here.

We differenciate the equations to get the equilibrium prices/profit maximising prices.

before merger the profit maximising prices were $60 for both p1 and p2 while after the merger, the total revenues also merge and now profit maximising prices are $70 for p1 and p2.

NOTE: WE HAVE ASSUMED p1=p2 as given in the question itself along with profit maximising equation of firm1 from which profit maximising equation of both the firms when together was calculated.


Related Solutions

Two firms compete in a market by selling differentiated products. The demand equations are : q1...
Two firms compete in a market by selling differentiated products. The demand equations are : q1 = 75 – p1 +p2/2 q2 = 75 – p2 +p1/2 Assume that each firm has a marginal cost and average costs of 0 What market model do we use if each firm competes by simultaneously choosing price Are the two goods substitutes? . Solve for firm 1’s best response function. Solve for the equilibrium price and quantity. Would firm 1 still be able...
In a market, there are two firms, firm A and firm B, producing differentiated products. Denoting...
In a market, there are two firms, firm A and firm B, producing differentiated products. Denoting the prices with pA and pB, firm A faces a demand given by qA(pA,pB) = 380−2pA +4 α pB, where 0 ≤ α < 1, and firm B faces a demand given by qB(pA,pB) = 180 − 2pB + pA. Each firm has a constant marginal cost, cA =cB =10and no fixed costs. (a)   Suppose that the firms compete in prices simultaneously. Find the...
firms with market power offer differentiated products in order to:
firms with market power offer differentiated products in order to:
Consider two firms that sell differentiated products and compete by choosing prices. Their demand functions are...
Consider two firms that sell differentiated products and compete by choosing prices. Their demand functions are Q1 = 72 – 3P1 + 2P2 and Q2 = 72 – 3P2 + 2P1 where P1 and P2 are the prices charged by firm 1 and 2, respectively, and Q1 and Q2 are the corresponding demands. All production costs are assumed to be zero. (a) Suppose the two firms set their prices simultaneously and non-cooperatively. Find the resulting Bertrand-Nash equilibrium. What price does...
Q2. Consider a Bertrand game with differentiated products in which two firms simul- taneously choose prices....
Q2. Consider a Bertrand game with differentiated products in which two firms simul- taneously choose prices. The marginal cost for each firm is zero and there are no fixed costs. The demand functions for each firm are: Q1 = 80 − 2P1 + 2P2, Q2 = 80 − 2P2 + 2P1. where P1 is the price set by firm 1, P2 is the price set by firm 2, Q1 is the quantity demanded of firm 1’s product and Q2 is...
Consider two identical firms competing as Cournot oligopolists in a market with demand p(Q)=100-0.5Q. Both firms...
Consider two identical firms competing as Cournot oligopolists in a market with demand p(Q)=100-0.5Q. Both firms have total costs,TC=10q where 10 is the marginal cost of production. ( Here Q represents total output in the market whereas q represents firm level output.) (b)   Now assume that the firms collude. They again play a one-shot game. What is the output that each firm should produce in order to sustain the collusion? Find the market price, and profits of each firm. Are...
Consider a market with two firms selling homogenous goods. Let the inverse demand curve be ?...
Consider a market with two firms selling homogenous goods. Let the inverse demand curve be ? = 1210 − 2(?1 + ?2) where p denotes market price and q1, q2 denotes output quantity from firm 1 and 2 respectively. The firms have identical constant marginal costs; c = 10. Assume first that the firms compete ala Cournot: a) Derive best response functions and illustrate equilibrium in a diagram b) Derive equilibrium quantities and prices c) Compute profit for each firm....
Consider two firms that provide a differentiated product, which they produce at the same constant marginal...
Consider two firms that provide a differentiated product, which they produce at the same constant marginal cost, MC = 3 (no fixed cost). The demand function for Firm 1 is q1 = 10 – p1 + 0.5p2 and for Firm 2 is q2 = 20 – p2 + 0.5p1, where p1 is Firm 1’s price and p2 is Firm 2’s price. a) Write the profit functions for these firms.( 8 marks) b) What are the equilibrium prices and quantities? (...
Two firms, A and B, produce complementary products. That is, rather than competing for the same...
Two firms, A and B, produce complementary products. That is, rather than competing for the same customers, demand for each firm’s output increases if the other firm produces more. Specifically, the inverse demand for Firm A’s product is given by pA = 110 − qA + qB. The inverse demand for Firm B’s product is given by pB = 50 − qB + qA. Both firms have marginal cost of 10 and choose quantities to maximize their profit. 5. If...
Two firms produce differentiated products and set prices to maximize their individual profits. Demand functions for...
Two firms produce differentiated products and set prices to maximize their individual profits. Demand functions for the firms are given by Q1 =64–4P1 +2P2 Q2 =50–5P2 +P1 where P1, P2, Q1, Q2, refer to prices and outputs of firms 1 and 2 respectively. Firm 1’s marginal cost is $5 while firm 2’s marginal cost is $4. Each firm has a fixed cost of $50. Assuming that the two firms decide on prices independently and simultaneously, calculate the best response function...
ADVERTISEMENT
ADVERTISEMENT
ADVERTISEMENT